21
Cardiovascular Blueprint Questions, Answers and Explanations

Cardiovascular Blueprint Questions, Answers and Explanations

Embed Size (px)

Citation preview

Page 1: Cardiovascular Blueprint Questions, Answers and Explanations

Cardiovascular Blueprint Questions, Answers and Explanations

Page 2: Cardiovascular Blueprint Questions, Answers and Explanations

Question 1

1. Your patient is a 45 year old male with a newly diagnosed right lower extremity DVT. Which of the following is the goal INR for him?

A. 1.5-2.5B. 2.0-3.0C. 3.0-4.0D. 3.5-4.5

Page 3: Cardiovascular Blueprint Questions, Answers and Explanations

Answer 1

1. B is the correct answer. Patients with their initial diagnosis of DVT need to be anti coagulated and have a goal INR of between 2.0-3.0 if coumadin is used. They should be anticoagulated for 3-6 months. Consider repeating the vascular study before discontinuing therapy. The second DVT should be anticoagulated for life and/or get a Greenfield filter.

Page 4: Cardiovascular Blueprint Questions, Answers and Explanations

Question 2

2. Your patient is a 54 year old male who presents with bilateral leg pain that has been present for several months. He is a smoker so your consider peripheral artery disease as a potential etiology of his symptoms. His ABI comes back at 1.5. Which of the following is true about this patient's findings?

A. This is a normal ABIB. This ABI is abnormal. C. This ABI is inaccurate. Smokers characteristically will have higher ABI's. This patient needs more sensitive studies such as angiographyD.We treat patients and not diagnostic tests. This patient should be started on Cilostazol empirically

Page 5: Cardiovascular Blueprint Questions, Answers and Explanations

Answer 2

2. Choice A is the correct answer. An ABI greater than 1.0 is considered normal and an ABI less than 1.0 is considered abnormal and suggestive of peripheral artery disease. ABI is fairly accurate. No clinical data suggests that smokers need angiography. It is a highly invasive procedure with complications and should be reserved for those high risk individuals are those with acute arterial occlusion or those where surgery may be necessary. Starting patients empirically on Cilostazol is not appropriate. These medications can cause bleeding and have side effects.

Page 6: Cardiovascular Blueprint Questions, Answers and Explanations

Question 3

3. Your patient is a 43 year old male that presents with chest pain, dizziness and shortness of breath. His past medical history is remarkable for CAD with 2 stents placed 6 months ago. His vital signs are 52/32, HR-154, SpO2 is 98%, RR-24 Temp-97.3. Which of the following is likely the cause of this patient's symptoms?

A. Volume DepletionB. Cardiogenic ShockC. Septic ShockD. CVA

Page 7: Cardiovascular Blueprint Questions, Answers and Explanations

Answer 3

3. Choice B is the correct answer. Cardiogenic shock is defined as widespread failure of inadequate tissue perfusion resulting in metabolic demands not being met. Myocardial ischemia is the primary cause. These patients will likely need to be placed on vasopressors. The main treatment is to treat the underlying myocardial ischemia first. Volume depletion can cause tachycardia and hypotension but typically will not cause you to have chest pain. Patients in septic shock will usually be febrile. A CVA typically will not be hypotensive or tachycardic.

Page 8: Cardiovascular Blueprint Questions, Answers and Explanations

Question 4

4. Your patient is a 64 year old male that presents with the EKG listed below. Which of the following is best therapeutic intervention?

A. DefibrillationB. LabetalolC. AdenosineD. Cardizem

Page 9: Cardiovascular Blueprint Questions, Answers and Explanations

Answer 4

4. Answer D is the correct answer. This patient has A-Fib with RVR. Defibrillation is reserved for Ventricular Tachycardia, Ventricular Fibrillation, or unstable A-Fib. This is a narrow complex tachycardia that is irregular. Adenosine or Labetalol would be appropriate for supraventricular tachycardia or sinus tachycardia.

Page 10: Cardiovascular Blueprint Questions, Answers and Explanations

Question 5

5. Your patient is a 59 year old female newly diagnosed with primary hypertension. Her blood pressure is 158/96. She has no other medical problems. Which of the following is the best choice for first line management for her?

A. ClonidineB. NorvascC. CardizemD. Hydrochlorothiazide

Page 11: Cardiovascular Blueprint Questions, Answers and Explanations

Answer 5

5. Choice D is the correct answer. According to the JNC guidelines diuretics a and beta blockers are considered first line for stage I hypertension. Medications such as clonidine that are centrally acting alpha blockers have some rebound effect if stopped abruptly. Calcium channel blockers such as norvasc and cardizem are considered second line.

Page 12: Cardiovascular Blueprint Questions, Answers and Explanations

Question 6

6. Your patient is a 84 year old female that presents with a new onset of systolic murmur best heard at the sternal border. She has been having near syncope getting progressively worse. She has no carotid bruits. Which of the following is the most likely diagnosis?

A. Aortic StenosisB. Aortic RegurgitationC. Carotid Artery StenosisD. Mitral Stenosis

Page 13: Cardiovascular Blueprint Questions, Answers and Explanations

Answer 6

6. Choice A is the correct answer. Aortic Stenosis presents with systolic murmur. Presenting symptoms are usually dizziness and sometimes chest pain. They also can present with CHF. Mitral Stenosis and Aortic Regurgitation are both diastolic murmurs so are incorrect. Carotid artery stenosis is not likely because the patient does not have any carotid bruits.

Page 14: Cardiovascular Blueprint Questions, Answers and Explanations

Question 77. Your patient is a 75 year old male that presents with sharp chest pain radiating through to his back. His vitals signs are BP 85/53, HR 110, RR 12, Temp 96.5. His Chem 7 is as follows Na 130, K 4.1, Cl-112, HCO3 22, BUN-36, Creatinine-2.5, Glucose 96. Which of the following is the best management option?

A. Order a Stat CTA of Chest with IV contrast to look for a dissectionB. Order a Stat Bedside ECHO C. Order a Transesophageal ECHO (TEE)D. Call the cardiothoracic surgeon and take the patient to the operating room

immediately.

Page 15: Cardiovascular Blueprint Questions, Answers and Explanations

Answer 7

7. Choice C is the correct answer. This patient with his presenting symptoms and impaired renal function needs a TEE. CT with IV contrast is contraindicated when the creatinine is greater than 2.0. The above two are the only two options for screening for a dissection. A normal ECHO is not going to be able give you any information rather the patient is dissecting or not. Taking the patient to surgery without a definitive diagnosis is dangerous and should not be done.

Page 16: Cardiovascular Blueprint Questions, Answers and Explanations

Question 8

8. Which of the following is the medication of choice for treating Prinzmetal’s Angina?

A. MetoprololB. LisinoprilC. VerapamilD. Nitroglycerin

Page 17: Cardiovascular Blueprint Questions, Answers and Explanations

Answer 8

8. Choice C is the correct answer. Prinzmetal’s angina or Variant angina is caused by spasm of the coronary arteries. It tends to occur at rest. Verapamil best targets it because it helps dilate arteries. Nitroglycerin has little activity on arteries. Metoprolol is useful in regular angina because it helps decrease myocardial demand by blocking the effect of catecholamines. Lisinopril helps with after load reduction.

Page 18: Cardiovascular Blueprint Questions, Answers and Explanations

Question 9

9. Which of the following is not recognized as a modifiable risk factor in the prevention of coronary disease?

A. DiabetesB. Alcohol intakeC. SmokingD. Hypertension

Page 19: Cardiovascular Blueprint Questions, Answers and Explanations

Answer 9

9. Choice B is the correct answer. Alcohol ingestion is not identified as a modifiable risk factor. In fact mild alcohol intake may be beneficial in helping heart disease. Diabetes, smoking, hypertension are modifiable risk factors.

Page 20: Cardiovascular Blueprint Questions, Answers and Explanations

Question 10

10. Your patient is a 53 year old male that presents with a dry cough and has been recently started on lisinopril for hypertension. Which of the following is the best explanation for the patient's symptoms?

A. Elevated Prostaglandin levelsB. Elevated Bradykinin levelsC. Elevated cyclooxygenase levelsD. Elevated catecholamines

Page 21: Cardiovascular Blueprint Questions, Answers and Explanations

Answer 10

10. Choice B is the correct answer. Elevated bradykinin levels are responsible for the dry cough in patients that take ACE inhibitors. It should be thought of as a class effect. Switching the patient to an Angiotensin Receptor Blocker would help this. There is no elevated bradykinin levels with that class. Prostaglandin levels are increased with ACE inhibitors and ARB’s but do not cause the dry cough. Elevated cyclooxygenase levels and catecholamine levels have nothing to do with the dry cough or ACE inhibitors.